Number Theory: Structures, Examples, and Problems


II Solutions, 9. Some Special Problems in Number Theory



Download 1,87 Mb.
Pdf ko'rish
bet120/125
Sana08.02.2022
Hajmi1,87 Mb.
#434761
1   ...   117   118   119   120   121   122   123   124   125
Bog'liq
Titu Andreescu, Dorin Andrica Number Theory Str

II Solutions, 9. Some Special Problems in Number Theory
(d) 2

a
<
m
. Rewrite
(
a
2
+
b
2
)/(
ab
+
1
)
=
m
2
as
b
2

m
2
ab
+
a
2

m
2
=
0
.
We know that
t
=
b
is a root of the quadratic equation
t
2

m
2
at
+
a
2

m
2
=
0
.
(
1
)
Thus
m
4
a
2
+
4
m
2

4
a
2
, the discriminant of the equation, must be a perfect
square. But
(
m
2
a
+
1
)
2
=
m
4
a
2
+
2
m
2
a
+
1
>
m
4
a
2
+
4
m
2

4
a
2
> (
m
2
a
)
2
for 2

a
<
m
. So the discriminant cannot be a perfect square, a contradiction.
(e)
a
>
m
. Again
t
=
b
is a root of (1). It is easy to check that
t
=
m
2
a

b
=
c
also satisfies the equation. We have
bc
=
a
2

m
2
>
0; since
b

0,
c
>
0. Since
a
>
0 and
c
>
0,
ac
+
1
>
0, we have
c
2
+
a
2
ca
+
1
=
m
2
.
Since
c
>
0,
b

a
, and
bc
=
a
2

m
2
<
a
2
, we have
c
<
a
. Thus
(
c
,
a
)
is a
valid pair. Also, it cannot be of the form
(
a
n
,
a
n
+
1
)
, or else
(
a
,
b
)
=
(
a
n
+
1
,
m
2
a
n
+
1

a
n
)
=
(
a
n
+
1
,
a
n
+
2
).
But then,
c
+
a
<
a
+
a

b
+
a
, as desired.
From the above, we see that our assumption is false. Therefore every pair
satisfying the original equation must be of the described form.
Second solution.
Note that if
a
=
0, then necessarily
b
=
m
, so
(
a
,
b
)
=
(
a
0
,
a
1
)
.
Also note that there is no solution with
a
,
b
both nonzero but of opposite signs.
Thus any other solution has
b

a
>
0. If
(
a
,
b
)
is a solution, then one easily
checks that
(
a
,
m
2
a

b
)
is again a solution. Since
a
>
0, this means we must have
m
2
a

b

0. Since
b
(
m
2
a

b
)
=
a
2

m
2
<
a
2
and
b
>
a
, we must also have
m
2
a

b
<
a
. Thus we have produced a smaller nonnegative solution. Because
we cannot reduce the solution indefinitely, reducing in this way must eventually
reach
(
0
,
m
)
. Therefore the original solution must have been
(
a
n
,
a
n
+
1
)
for some
n
.
Problem 9.3.14.
Let b
,
c be positive integers, and define the sequence a
1
,
a
2
, . . .
by a
1
=
b, a
2
=
c, and
a
n
+
2
= |
3
a
n
+
1

2
a
n
|
for n

1
. Find all such
(
b
,
c
)
for which the sequence a
1
,
a
2
, . . .
has only a finite
number of composite terms.
(2002 Bulgarian Mathematical Olympiad)


9.3. Sequences of Integers
341
Solution.
The only solutions are
(
p
,
p
)
for
p
not composite,
(
2
p
,
p
)
for
p
not
composite, and
(
7
,
4
)
.
The sequence
a
1
,
a
2
, . . .
cannot be strictly decreasing because each
a
n
is a
positive integer, so there exists a smallest
k

1 such that
a
k
+
1

a
k
. Define a
new sequence
b
1
,
b
2
, . . .
by
b
n
=
a
n
+
k

1
, so
b
2

b
1
,
b
n
+
2
= |
3
b
n
+
1

2
b
n
|
for
n

1, and
b
1
,
b
2
, . . .
has only a finite number of composite terms. Now, if
b
n
+
1

b
n
,
b
n
+
2
= |
3
b
n
+
1

2
b
n
| =
3
b
n
+
1

2
b
n
=
b
n
+
1
+
2
(
b
n
+
1

b
n
)

b
n
+
1
,
so by induction
b
n
+
2
=
3
b
n
+
1

2
b
n
for
n

1.
Using the general theory of linear recursion relations (a simple induction proof
also suffices), we have
b
n
=
A
·
2
n

1
+
B
for
n

1, where
A
=
b
2

b
1
,
B
=
2
b
1

b
2
. Suppose (for contradiction) that
A
=
0. Then
b
n
is an increasing sequence, and since it contains only finitely
many composite terms,
b
n
=
p
for some prime
p
>
2 and some
n

1. But then
b
n
+
l
(
p

1
)
would be divisible by
p
and thus composite for
l

1, because
b
n
+
l
(
p

1
)
=
A
·
2
n

1
·
2
l
(
p

1
)
+
B

A
·
2
n

1
+
B

b
n

0
(
mod
p
)
by Fermat’s little theorem. This is a contradiction, so
A
=
0 and
b
n
=
b
1
for
n

1. Therefore
b
1
is not composite; let
b
1
=
p
, where
p
=
1 or
p
is prime.
We now return to the sequence
a
1
,
a
2
, . . .
, and consider different possible
values of
k
. If
k
=
1, we have
a
1
=
b
1
=
b
2
=
a
2
=
p
, so
b
=
c
=
p
for
p
not composite. If
k
>
1, consider that
a
k

1
>
a
k
by the choice of
k
, but
a
k
+
1
= |
3
a
k

2
a
k

1
|
, and
a
k
+
1
=
b
2
=
b
1
=
a
k
, so
a
k
+
1
=
2
a
k

1

3
a
k
, and
thus
a
k

1
=
2
p
. For
k
=
2, this means that
b
=
2
p
,
c
=
p
for
p
not composite.
If
k
>
2, the same approach yields
a
k

2
=
3
a
k

1
±
a
k
2
=
7
2
p
or
5
2
p
,
so
p
=
2. For
k
=
3, this gives solutions
b
=
7 or
b
=
5,
c
=
4. Because
3
·
5
±
4
2
and
3
·
7
±
4
2
are not integers, there are no solutions for
k
>
3.
Remark.
The reader may try to prove the following more general statement:
Let
f

Z
[
X
1
, . . . ,
X
k
]
be a polynomial and F
(
n
)
=
f
(
n
,
2
n
,
3
n
, . . . , (
k

1
)
n
)
,
n

1
. If
lim
n
→∞
F
(
n
)
= ∞
, then the set of primes dividing the terms of the
sequence
(
F
(
n
))
n

1
is infinite.


342
II Solutions, 9. Some Special Problems in Number Theory
9.3.3
Nonstandard Sequences of Integers
Problem 9.3.21.
Let
{
a
n
}
be a sequence of integers such that for n

1
,
(
n

1
)
a
n
+
1
=
(
n
+
1
)
a
n

2
(
n

1
).
If
2000
divides a
1999
, find the smallest n

2
such that
2000
divides a
n
.
(1999 Bulgarian Mathematical Olympiad)
Solution.
First, we note that the sequence
a
n
=
2
n

2 works. Then writing
b
n
=
a
n

(
2
n

2
)
gives the recursion
(
n

1
)
b
n
+
1
=
(
n
+
1
)
b
n
.
For
n

2, observe that
b
n
=
b
2
n

1
k
=
2
k
+
1
k

1
=
b
2
n
k
=
3
k
n

2
k
=
1
k
=
n
(
n

1
)
2
b
2
.
Thus when
n

2, the solution to the original equation of the form
a
n
=
2
(
n

1
)
+
n
(
n

1
)
2
c
for some constant
c
. Plugging in
n
=
2 shows that
c
=
a
2

2 is an integer.
Now, because 2000
|
a
1999
we have
2
(
1999

1
)
+
1999
·
1998
2
c

0
(
mod 2000
).
This implies

4
+
1001
c

0
(
mod 2000
),
hence
c

4
(
mod 2000
).
Then 2000
|
a
n
exactly when
2
(
n

1
)
+
2
n
(
n

1
)

0
(
mod 2000
)

(
n

1
)(
n
+
1
)

0
(
mod 1000
).
The number
(
n

1
)(
n
+
1
)
is divisible by 8 exactly when
n
is odd, and it is divisible
by 125 exactly when either
n

1 or
n
+
1 is divisible by 125. The smallest
n

2
satisfying these requirements is
n
=
249.
Problem 9.3.22.
The sequence
(
a
n
)
n

0
is defined by a
0
=
1
, a
1
=
3
, and
a
n
+
2
=
a
n
+
1
+
9
a
n
if n is even
,
9
a
n
+
1
+
5
a
n
if n is odd
.
Prove that
(a)
"
2000
k
=
1995
a
2
k
is divisible by
20
,
(b) a
2
n
+
1
is not a perfect square for any n
=
0
,
1
,
2
, . . .
.
(1995 Vietnamese Mathematical Olympiad)


9.3. Sequences of Integers
343
Solution.
(a) We will first prove that the sum is divisible by 4, then by 5. Note
that
a
n
+
2

a
n
+
1
+
a
n
(
mod 4
)
whether
n
is odd or even. The sequence modulo
4 thus proceeds 1, 3, 0, 3, 3, 2, 1, 3,
. . .
in a cycle of 6, so the sum of squares
of any six consecutive terms is congruent to 1
2
+
3
2
+
0
2
+
3
2
+
3
2
+
2
2

0
(
mod 4
)
.
Now let us work modulo 5, in which case
a
n
+
2

a
n
+
1
+
4
a
n
if
n
is even and
a
n
+
2

4
a
n
+
1
if
n
is odd. Hence the sequence modulo 5 proceeds 1, 3, 2, 3, 1, 4,
3, 2, 4, 1, 2, 3,
. . .
in a cycle of 8 beginning with
a
2
. This means that
a
2
1995
+· · ·+
a
2
2000

a
2
3
+· · ·+
a
2
8

3
2
+
1
2
+
4
2
+
3
2
+
2
2
+
4
2

0
(
mod 5
).
(b) From part (a) we have
a
2
n
+
1

2 or 3
(
mod 4
)
, which implies that
a
2
n
+
1
is not a square.
Problem 9.3.23.
Prove that for any natural number a
1
>
1
, there exists an in-
creasing sequence of natural numbers a
1
,
a
2
, . . .
such that a
2
1
+
a
2
2
+ · · · +
a
2
k
is
divisible by a
1
+
a
2
+ · · · +
a
k
for all k

1
.
(1995 Russian Mathematical Olympiad)
Solution.
We will prove in fact that any finite sequence
a
1
, . . . ,
a
k
with the prop-
erty can be extended by a suitable
a
k
+
1
. Let
s
k
=
a
1
+ · · · +
a
k
and
t
k
=
a
2
1
+ · · · +
a
2
k
. Then we are seeking
a
k
+
1
such that
a
k
+
1
+
s
k
|
a
2
k
+
1
+
t
k
. This is
clearly equivalent to
a
k
+
1
+
s
k
|
s
2
k
+
t
k
. Why not, then, choose
a
k
+
1
=
s
2
k

s
k
+
t
k
?
Certainly this is greater than
a
k
and ensures that the desired property is satisfied.
Problem 9.3.24.
The sequence a
0
,
a
1
,
a
2
, . . .
satisfies
a
m
+
n
+
a
m

n
=
1
2
(
a
2
m
+
a
2
n
)
for all nonnegative integers m and n with m

n. If a
1
=
1
, determine a
n
.
(1995 Russian Mathematical Olympiad)
Solution.
The relations
a
2
m
+
a
2
m
=
2
(
a
2
m
+
a
0
)
=
4
(
a
m
+
a
m
)
imply
a
2
m
=
4
a
m
,
as well as
a
0
=
0. Thus we compute
a
2
=
4,
a
4
=
16. Also,
a
1
+
a
3
=
(
a
2
+
a
4
)/
2
=
10 so
a
3
=
9. At this point we guess that
a
i
=
i
2
for all
i

1.
We prove our guess by induction on
i
. Suppose that
a
j
=
j
2
for
j
<
i
. Then
the given equation with
m
=
i

1,
j
=
1 gives
a
i
=
1
2
(
a
2
i

2
+
a
2
)

a
i

2
=
2
a
i

1
+
2
a
1

a
i

2
=
2
(
i
2

2
i
+
1
)
+
2

(
i
2

4
i
+
4
)
=
i
2
.
Problem 9.3.25.
The sequence of real numbers a
1
,
a
2
,
a
3
, . . .
satisfies the initial
conditions a
1
=
2
, a
2
=
500
, a
3
=
2000
as well as the relation
a
n
+
2
+
a
n
+
1
a
n
+
1
+
a
n

1
=
a
n
+
1
a
n

1


344
II Solutions, 9. Some Special Problems in Number Theory
for n
=
2
,
3
,
4
, . . .
Prove that all the terms of this sequence are positive integers
and that
2
2000
divides the number a
2000
.
(1999 Slovenian Mathematical Olympiad)
First solution.
From the recursion relation it follows that
a
n
+
2
a
n

1
=
a
2
n
+
1
for
n
=
2
,
3
, . . .
. No term of our sequence can equal 0, and hence it is possible to
write
a
n
+
2
a
n
+
1
a
n
=
a
n
+
1
a
n
a
n

1
(
1
)
for
n
=
2
,
3
, . . .
. It follows by induction that the value of the expression
a
n
+
1
a
n
a
n

1
is constant, namely equal to
a
3
/
a
2
a
1
=
2. Thus
a
n
+
2
=
2
a
n
a
n
+
1
and all terms of
the sequence are positive integers.
From this new relation, we also know that
a
n
+
1
/
a
n
is an even integer for all
positive integers
n
. Write
a
2000
=
a
2000
a
1999
a
1999
a
1998
· · ·
a
2
a
1
a
1
.
In this product each of the 1999 fractions is divisible by 2, and
a
1
=
2 is even
as well. Thus
a
2000
is indeed divisible by 2
2000
.
Second solution.
Note that
a
n
=
2
F
n
+
2

1
·
5
3
F
n

1
, proved by induction by using
equation (1) in the previous solution, where
F
n
are the Fibonacci numbers,
n

1.
Hence the divisibility is a consequence of
F
2002

2001.
Problem 9.3.26.
Let k be a fixed positive integer. We define the sequence a
1
,
a
2
,
. . .
by a
1
=
k
+
1
and the recursion a
n
+
1
=
a
2
n

ka
n
+
k for n

1
. Prove that
a
m
and a
n
are relatively prime for distinct positive integers m and n.
First solution.
We claim that
a
n
=
n

1
i
=
0
a
i
+
k
,
n
>
0
,
assuming that
a
0
=
1. Since
a
j
+
1

k
=
a
j
(
a
j

k
)
, we have
a
n

k
=
n

1
j
=
1
a
j
+
1

k
a
j

k
=
n

1
j
=
1
a
j
,
which is what we wanted.
Therefore, we have that
a
n

k
(
mod
a
i
)
for
i
<
n
. Hence, if there exist
integers
d
>
1,
x
,
y

1 such that
d
|
a
x
and
d
|
a
y
,
d
divides
k
. We now show


9.3. Sequences of Integers
345
that for
i
>
0,
a
i

1
(
mod
k
)
by induction on
i
. For the base case,
a
1
=
k
+
1

1
(
mod
k
)
. Now assume that
a
i

1
(
mod
k
)
. Then,
a
i
+
1

a
2
i

ka
i
+
k

a
2
i

1
(
mod
k
)
. Thus, because all common divisors
d
of
a
x
and
a
y
must be divisors
of
k
, we have
a
x

1
(
mod
d
)
and
a
y

1
(
mod
d
)
. Therefore, no such divisors
exist and
a
i
is relatively prime to
a
j
for all
i
,
j
>
0, as desired.
Second solution.
First, by induction on
n
, it follows that
a
n

1
(
mod
k
)
for
all
n
. Then it follows by induction on
m
that
a
m

k
(
mod
a
n
)
for all
m
>
n
.
Therefore for
m
>
n
we have gcd
(
a
m
,
a
n
)
=
gcd
(
k
,
a
n
)
=
gcd
(
k
,
1
)
=
1.
Problem 9.3.27.
Suppose the sequence of nonnegative integers a
1
, a
2
, . . .
, a
1997
satisfies
a
i
+
a
j

a
i
+
j

a
i
+
a
j
+
1
for all i
,
j

1
with i
+
j

1997
. Show that there exists a real number x such
that a
n

nx
for all
1

n

1997
.
(1997 USA Mathematical Olympiad)
Solution.
Any
x
that lies in all of the half-open intervals
I
n
=
1
a
n
n
,
a
n
+
1
n
,
n
=
1
,
2
, . . . ,
1997
,
will have the desired property. Let
L
=
max
1

n

1997
a
n
n
=
a
p
p
and
U
=
min
1

n

1997
a
n
+
1
n
=
a
q
+
1
q
.
We shall prove that
a
n
n
<
a
m
+
1
m
,
or equivalently,
ma
n
<
n
(
a
m
+
1
)
(

)
for all
m
,
n
ranging from 1 to 1997. Then
L
<
U
, since
L

U
implies that
(

)
is
violated when
n
=
p
and
m
=
q
. Any point
x
in
[
L
,
U
)
has the desired property.
We prove
(

)
for all
m
,
n
ranging from 1 to 1997 by strong induction. The
base case
m
=
n
=
1 is trivial. The induction step splits into three cases. If
m
=
n
, then
(

)
certainly holds. If
m
>
n
, then the induction hypothesis gives
(
m

n
)
a
n
<
n
(
a
m

n
+
1
)
, and adding
n
(
a
m

n
+
a
n
)

na
m
yields
(

)
. If
m
<
n
,
then the induction hypothesis yields
ma
n

m
< (
n

m
)(
a
m
+
1
)
, and adding
ma
n

m
(
a
m
+
a
n

m
+
1
)
gives
(

)
.
Problem 9.3.28.
The sequence
{
a
n
}
is given by the following relation:
a
n
+
1
=
a
n

1
2
,
if a
n

1
,
2
a
n
1

a
n
,
if a
n
<
1
.


346
II Solutions, 9. Some Special Problems in Number Theory
Given that a
0
is a positive integer, a
n
=
2
for each n
=
1
,
2
, . . . ,
2001
, and
a
2002
=
2
, find a
0
.
(2002 St. Petersburg City Mathematical Olympiad)
Solution.
Answer:
a
0
=
3
·
2
2002

1.
Suppose we are given
a
n
+
1
. Then there are exactly two possibilities for
a
n
. If
a
n

1, then the first rule gives
a
n
=
2
a
n
+
1
+
1 (which is at least 1 as required). If
a
n
<
1, then the second rule gives
a
n
=
a
n
+
1
a
n
+
1
+
2
(which is less than 1 as required).
Thus the problem amounts to the following: Start with
a
2002
=
2 and repeatedly
apply one of the two transformations
a
n
=
2
a
n
+
1
+
1
,
a
n
+
1
a
n
+
1
+
2
.
Suppose you never again get
a
n
=
2 and suppose
a
0
is an integer. Then what
is
a
0
?
If we apply the first rule 2002 times, then
a
n
+
1 doubles every step (and in
Download 1,87 Mb.

Do'stlaringiz bilan baham:
1   ...   117   118   119   120   121   122   123   124   125




Ma'lumotlar bazasi mualliflik huquqi bilan himoyalangan ©hozir.org 2024
ma'muriyatiga murojaat qiling

kiriting | ro'yxatdan o'tish
    Bosh sahifa
юртда тантана
Боғда битган
Бугун юртда
Эшитганлар жилманглар
Эшитмадим деманглар
битган бодомлар
Yangiariq tumani
qitish marakazi
Raqamli texnologiyalar
ilishida muhokamadan
tasdiqqa tavsiya
tavsiya etilgan
iqtisodiyot kafedrasi
steiermarkischen landesregierung
asarlaringizni yuboring
o'zingizning asarlaringizni
Iltimos faqat
faqat o'zingizning
steierm rkischen
landesregierung fachabteilung
rkischen landesregierung
hamshira loyihasi
loyihasi mavsum
faolyatining oqibatlari
asosiy adabiyotlar
fakulteti ahborot
ahborot havfsizligi
havfsizligi kafedrasi
fanidan bo’yicha
fakulteti iqtisodiyot
boshqaruv fakulteti
chiqarishda boshqaruv
ishlab chiqarishda
iqtisodiyot fakultet
multiservis tarmoqlari
fanidan asosiy
Uzbek fanidan
mavzulari potok
asosidagi multiservis
'aliyyil a'ziym
billahil 'aliyyil
illaa billahil
quvvata illaa
falah' deganida
Kompyuter savodxonligi
bo’yicha mustaqil
'alal falah'
Hayya 'alal
'alas soloh
Hayya 'alas
mavsum boyicha


yuklab olish